Suppose $BA$ is nilpotent, is it always true that $AB$ is nilpotent?Nilpotent matrices with same minimal polynomial and nullityIs the zero matrix the only symmetric, nilpotent matrix with real values?Smoothness of a MatrixSuppose $A^2B+BA^2=2ABA$.Prove that there exists a positive integer $k$ such that $(AB-BA)^k=0$.Product of nilpotent matrices is nilpotentProduct of nilpotent matrices invertibleExample of non-nilpotent product of nilpotent matricesUnitriangular group $UT_n(Bbb Z)$ is nilpotent with class $n$Proving an operator is nilpotentNilpotent matrix sets

How can a day be of 24 hours?

Is it possible to create a QR code using text?

How to compactly explain secondary and tertiary characters without resorting to stereotypes?

how do we prove that a sum of two periods is still a period?

How do I exit BASH while loop using modulus operator?

What is the most common color to indicate the input-field is disabled?

What reasons are there for a Capitalist to oppose a 100% inheritance tax?

Placement of More Information/Help Icon button for Radio Buttons

Fair gambler's ruin problem intuition

What is required to make GPS signals available indoors?

How to stretch the corners of this image so that it looks like a perfect rectangle?

Can compressed videos be decoded back to their uncompresed original format?

What is an equivalently powerful replacement spell for the Yuan-Ti's Suggestion spell?

How does a dynamic QR code work?

Bullying boss launched a smear campaign and made me unemployable

How do conventional missiles fly?

"the same as" in a sentence

Why didn't Boeing produce its own regional jet?

Is there an expression that means doing something right before you will need it rather than doing it in case you might need it?

Should I tell management that I intend to leave due to bad software development practices?

How dangerous is XSS

Is it a bad idea to plug the other end of ESD strap to wall ground?

Could the museum Saturn V's be refitted for one more flight?

Finitely generated matrix groups whose eigenvalues are all algebraic



Suppose $BA$ is nilpotent, is it always true that $AB$ is nilpotent?


Nilpotent matrices with same minimal polynomial and nullityIs the zero matrix the only symmetric, nilpotent matrix with real values?Smoothness of a MatrixSuppose $A^2B+BA^2=2ABA$.Prove that there exists a positive integer $k$ such that $(AB-BA)^k=0$.Product of nilpotent matrices is nilpotentProduct of nilpotent matrices invertibleExample of non-nilpotent product of nilpotent matricesUnitriangular group $UT_n(Bbb Z)$ is nilpotent with class $n$Proving an operator is nilpotentNilpotent matrix sets













2












$begingroup$


Given matrices $A$ and $B$ such that $BA$ is nilpotent. Is it true that $AB$ is nilpotent?



My thoughts so far:



$BA$ nilpotent $Rightarrow (BA)^k = 0$ for some positive integer k.



$(BA)^k = (BA)(BA)....(BA) = BABA...BA = B(AB)(AB)....(AB)A = B(AB)^k-1A = 0$



But I have no idea how to proceed from here. If its not true, is there any counterexample?










share|cite|improve this question











$endgroup$
















    2












    $begingroup$


    Given matrices $A$ and $B$ such that $BA$ is nilpotent. Is it true that $AB$ is nilpotent?



    My thoughts so far:



    $BA$ nilpotent $Rightarrow (BA)^k = 0$ for some positive integer k.



    $(BA)^k = (BA)(BA)....(BA) = BABA...BA = B(AB)(AB)....(AB)A = B(AB)^k-1A = 0$



    But I have no idea how to proceed from here. If its not true, is there any counterexample?










    share|cite|improve this question











    $endgroup$














      2












      2








      2





      $begingroup$


      Given matrices $A$ and $B$ such that $BA$ is nilpotent. Is it true that $AB$ is nilpotent?



      My thoughts so far:



      $BA$ nilpotent $Rightarrow (BA)^k = 0$ for some positive integer k.



      $(BA)^k = (BA)(BA)....(BA) = BABA...BA = B(AB)(AB)....(AB)A = B(AB)^k-1A = 0$



      But I have no idea how to proceed from here. If its not true, is there any counterexample?










      share|cite|improve this question











      $endgroup$




      Given matrices $A$ and $B$ such that $BA$ is nilpotent. Is it true that $AB$ is nilpotent?



      My thoughts so far:



      $BA$ nilpotent $Rightarrow (BA)^k = 0$ for some positive integer k.



      $(BA)^k = (BA)(BA)....(BA) = BABA...BA = B(AB)(AB)....(AB)A = B(AB)^k-1A = 0$



      But I have no idea how to proceed from here. If its not true, is there any counterexample?







      linear-algebra






      share|cite|improve this question















      share|cite|improve this question













      share|cite|improve this question




      share|cite|improve this question








      edited 2 days ago









      Asaf Karagila

      307k33440773




      307k33440773










      asked 2 days ago









      aaaaaa

      374




      374




















          3 Answers
          3






          active

          oldest

          votes


















          4












          $begingroup$

          $(BA)^k = (BA)(BA)....(BA) = BABA...BA =0$



          Multiply from the left by $A$ and from the right by $B$, you'll get $$ A.BABA...BA.B= (AB)(AB)...(AB)=(AB)^k+1=0$$






          share|cite|improve this answer









          $endgroup$




















            5












            $begingroup$

            You just have to replace $A$ by $B$ in your computation. Then $(AB)^k +1= A(BA)^k B$. I assume you can finish from here.






            share|cite|improve this answer









            $endgroup$




















              2












              $begingroup$

              You can multiply by A on the left side and B on the right side, and you get $(AB)^k+1 = 0$






              share|cite|improve this answer









              $endgroup$













                Your Answer





                StackExchange.ifUsing("editor", function ()
                return StackExchange.using("mathjaxEditing", function ()
                StackExchange.MarkdownEditor.creationCallbacks.add(function (editor, postfix)
                StackExchange.mathjaxEditing.prepareWmdForMathJax(editor, postfix, [["$", "$"], ["\\(","\\)"]]);
                );
                );
                , "mathjax-editing");

                StackExchange.ready(function()
                var channelOptions =
                tags: "".split(" "),
                id: "69"
                ;
                initTagRenderer("".split(" "), "".split(" "), channelOptions);

                StackExchange.using("externalEditor", function()
                // Have to fire editor after snippets, if snippets enabled
                if (StackExchange.settings.snippets.snippetsEnabled)
                StackExchange.using("snippets", function()
                createEditor();
                );

                else
                createEditor();

                );

                function createEditor()
                StackExchange.prepareEditor(
                heartbeatType: 'answer',
                autoActivateHeartbeat: false,
                convertImagesToLinks: true,
                noModals: true,
                showLowRepImageUploadWarning: true,
                reputationToPostImages: 10,
                bindNavPrevention: true,
                postfix: "",
                imageUploader:
                brandingHtml: "Powered by u003ca class="icon-imgur-white" href="https://imgur.com/"u003eu003c/au003e",
                contentPolicyHtml: "User contributions licensed under u003ca href="https://creativecommons.org/licenses/by-sa/3.0/"u003ecc by-sa 3.0 with attribution requiredu003c/au003e u003ca href="https://stackoverflow.com/legal/content-policy"u003e(content policy)u003c/au003e",
                allowUrls: true
                ,
                noCode: true, onDemand: true,
                discardSelector: ".discard-answer"
                ,immediatelyShowMarkdownHelp:true
                );



                );













                draft saved

                draft discarded


















                StackExchange.ready(
                function ()
                StackExchange.openid.initPostLogin('.new-post-login', 'https%3a%2f%2fmath.stackexchange.com%2fquestions%2f3169534%2fsuppose-ba-is-nilpotent-is-it-always-true-that-ab-is-nilpotent%23new-answer', 'question_page');

                );

                Post as a guest















                Required, but never shown

























                3 Answers
                3






                active

                oldest

                votes








                3 Answers
                3






                active

                oldest

                votes









                active

                oldest

                votes






                active

                oldest

                votes









                4












                $begingroup$

                $(BA)^k = (BA)(BA)....(BA) = BABA...BA =0$



                Multiply from the left by $A$ and from the right by $B$, you'll get $$ A.BABA...BA.B= (AB)(AB)...(AB)=(AB)^k+1=0$$






                share|cite|improve this answer









                $endgroup$

















                  4












                  $begingroup$

                  $(BA)^k = (BA)(BA)....(BA) = BABA...BA =0$



                  Multiply from the left by $A$ and from the right by $B$, you'll get $$ A.BABA...BA.B= (AB)(AB)...(AB)=(AB)^k+1=0$$






                  share|cite|improve this answer









                  $endgroup$















                    4












                    4








                    4





                    $begingroup$

                    $(BA)^k = (BA)(BA)....(BA) = BABA...BA =0$



                    Multiply from the left by $A$ and from the right by $B$, you'll get $$ A.BABA...BA.B= (AB)(AB)...(AB)=(AB)^k+1=0$$






                    share|cite|improve this answer









                    $endgroup$



                    $(BA)^k = (BA)(BA)....(BA) = BABA...BA =0$



                    Multiply from the left by $A$ and from the right by $B$, you'll get $$ A.BABA...BA.B= (AB)(AB)...(AB)=(AB)^k+1=0$$







                    share|cite|improve this answer












                    share|cite|improve this answer



                    share|cite|improve this answer










                    answered 2 days ago









                    Fareed AFFareed AF

                    692112




                    692112





















                        5












                        $begingroup$

                        You just have to replace $A$ by $B$ in your computation. Then $(AB)^k +1= A(BA)^k B$. I assume you can finish from here.






                        share|cite|improve this answer









                        $endgroup$

















                          5












                          $begingroup$

                          You just have to replace $A$ by $B$ in your computation. Then $(AB)^k +1= A(BA)^k B$. I assume you can finish from here.






                          share|cite|improve this answer









                          $endgroup$















                            5












                            5








                            5





                            $begingroup$

                            You just have to replace $A$ by $B$ in your computation. Then $(AB)^k +1= A(BA)^k B$. I assume you can finish from here.






                            share|cite|improve this answer









                            $endgroup$



                            You just have to replace $A$ by $B$ in your computation. Then $(AB)^k +1= A(BA)^k B$. I assume you can finish from here.







                            share|cite|improve this answer












                            share|cite|improve this answer



                            share|cite|improve this answer










                            answered 2 days ago









                            Severin SchravenSeverin Schraven

                            6,7802936




                            6,7802936





















                                2












                                $begingroup$

                                You can multiply by A on the left side and B on the right side, and you get $(AB)^k+1 = 0$






                                share|cite|improve this answer









                                $endgroup$

















                                  2












                                  $begingroup$

                                  You can multiply by A on the left side and B on the right side, and you get $(AB)^k+1 = 0$






                                  share|cite|improve this answer









                                  $endgroup$















                                    2












                                    2








                                    2





                                    $begingroup$

                                    You can multiply by A on the left side and B on the right side, and you get $(AB)^k+1 = 0$






                                    share|cite|improve this answer









                                    $endgroup$



                                    You can multiply by A on the left side and B on the right side, and you get $(AB)^k+1 = 0$







                                    share|cite|improve this answer












                                    share|cite|improve this answer



                                    share|cite|improve this answer










                                    answered 2 days ago









                                    GatgatGatgat

                                    1775




                                    1775



























                                        draft saved

                                        draft discarded
















































                                        Thanks for contributing an answer to Mathematics Stack Exchange!


                                        • Please be sure to answer the question. Provide details and share your research!

                                        But avoid


                                        • Asking for help, clarification, or responding to other answers.

                                        • Making statements based on opinion; back them up with references or personal experience.

                                        Use MathJax to format equations. MathJax reference.


                                        To learn more, see our tips on writing great answers.




                                        draft saved


                                        draft discarded














                                        StackExchange.ready(
                                        function ()
                                        StackExchange.openid.initPostLogin('.new-post-login', 'https%3a%2f%2fmath.stackexchange.com%2fquestions%2f3169534%2fsuppose-ba-is-nilpotent-is-it-always-true-that-ab-is-nilpotent%23new-answer', 'question_page');

                                        );

                                        Post as a guest















                                        Required, but never shown





















































                                        Required, but never shown














                                        Required, but never shown












                                        Required, but never shown







                                        Required, but never shown

































                                        Required, but never shown














                                        Required, but never shown












                                        Required, but never shown







                                        Required, but never shown







                                        Popular posts from this blog

                                        Sum ergo cogito? 1 nng

                                        419 nièngy_Soadمي 19bal1.5o_g

                                        Queiggey Chernihivv 9NnOo i Zw X QqKk LpB